Neuro Question collection

Réussis tes devoirs et examens dès maintenant avec Quizwiz!

When providing discharge teaching for a client with multiple sclerosis (MS), the nurse should include which instruction? "Avoid taking daytime naps." "Avoid hot baths and showers." "Limit your fruit and vegetable intake." "Restrict fluid intake to 1,500 ml/day."

"Avoid hot baths and showers." Explanation: The nurse should instruct a client with MS to avoid hot baths and showers because they may exacerbate the disease. The nurse should encourage daytime naps because fatigue is a common symptom of MS. A client with MS doesn't require food or fluid restrictions.

A client with Guillain-Barré syndrome has paralysis affecting the respiratory muscles and requires mechanical ventilation. When the client asks the nurse about the paralysis, how should the nurse respond? "The paralysis caused by this disease is temporary." "You'll be permanently paralyzed; however, you won't have any sensory loss." "It must be hard to accept the permanency of your paralysis." "You'll first regain use of your legs and then your arms."

"The paralysis caused by this disease is temporary." Explanation: The nurse should inform the client that the paralysis that accompanies Guillain-Barré syndrome is only temporary. Return of motor function begins proximally and extends distally in the legs.

Which Glasgow Coma Scale score is indicative of a severe head injury? 7 9 11 13

7 Explanation: A score between 3 and 8 is generally accepted as indicating a severe head injury.

The nurse is assigned to care for clients with SCI on a rehabilitation unit. Which signs does the nurse recognize as clinical manifestations of autonomic dysreflexia? Select all that apply. Hypertension Tachycardia Fever Diaphoresis Nasal congestion

Hypertension Diaphoresis Nasal congestion Explanation: Hypertension and diaphoresis are signs of autonomic dysreflexia. Nasal congestion often accompanies autonomic dysreflexia. Bradycardia, not tachycardia, occurs with autonomic dysreflexia. Although the client may be diaphoretic, a fever does not accompany this condition.

A nurse is caring for a client who has returned to his room after a carotid endarterectomy. Which action should the nurse take first? Ask the client if he has trouble breathing. Take the client's blood pressure. Ask the client if he has a headache. Place antiembolism stockings on the client.

Ask the client if he has trouble breathing. Explanation: The nurse should first assess the client's breathing. A complication of a carotid endarterectomy is an incisional hematoma, which could compress the trachea causing breathing difficulty for the client. Although the other measures are important actions, they aren't the nurse's top priority.

Damage to the brain from traumatic injury can be divided into primary and secondary injuries. Which of the following is cause of a secondary injury associated with brain injury? Select all that apply. Cerebral edema Ischemia Infection Seizures Hyperthermia

Cerebral edema Ischemia Infection Seizures Hyperthermia Explanation: Secondary injury evolves over the ensuing hours and days after the initial injury and can be due to cerebral edema, ischemia, seizures, infection, hyperthermia, hypovolemia, and hypoxia.

The nurse is caring for a client who has been admitted with a head injury and continually assesses for signs of increasing intracranial pressure (ICP). The earliest sign of increasing ICP is Change in level of consciousness Widening pulse pressure Slowing of heart rate Elevation of systolic blood pressure

Change in level of consciousness Explanation: The earliest sign of increasing ICP is a change in level of consciousness. Other early indicators are slowing of speech and delay in response to verbal suggestions. The other three choices are all parts of a clinical phenomenon known as the Cushing's response, which is a late sign of increasing ICP.

After a stroke, a client is admitted to the facility. The client has left-sided weakness and an absent gag reflex. He's incontinent and has a tarry stool. His blood pressure is 90/50 mm Hg, and his hemoglobin is 10 g. Which nursing intervention is a priority for this client? Checking stools for occult blood Performing range-of-motion (ROM) exercises on the left side Keeping skin clean and dry Elevating the head of the bed to 30 degrees

Elevating the head of the bed to 30 degrees Explanation: Because the client's gag reflex is absent, elevating the head of the bed to 30 degrees helps minimize the client's risk of aspiration. Checking the stools, performing ROM exercises, and keeping the skin clean and dry are important, but preventing aspiration through positioning is the priority.

The school nurse has been called to the football field where player is immobile on the field after landing awkwardly on his head during a play. While awaiting an ambulance, what action should the nurse perform? Ensure that the player is not moved. Obtain the player's vital signs, if possible. Perform a rapid assessment of the player's range of motion. Assess the player's reflexes.

Ensure that the player is not moved. Explanation: At the scene of the injury, the client must be immobilized on a spinal (back) board, with the head and neck maintained in a neutral position, to prevent an incomplete injury from becoming complete. This is a priority over determining the client's vital signs. It would be inappropriate to test ROM or reflexes.

A client has been diagnosed as having global aphasia. The nurse recognizes that the client will be unable to perform which action? Comprehend spoken words Form words that are understandable Form words that are understandable or comprehend spoken words Speak at all

Form words that are understandable or comprehend spoken words Explanation: Global aphasia is a combination of expressive and receptive aphasia and presents a tremendous challenge to the nurse to effectively communicate with the client. In receptive and expressive aphasia, the client is unable to form words that are understandable. The client who is unable to speak at all is referred to as mute.

A client exhibiting an altered level of consciousness (LOC) due to blunt force trauma to the head is admitted to the ED. The nurse should gauge the client's LOC on the results of what diagnostic tool? Monro-Kellie hypothesis Glasgow Coma scale Cranial nerve function Mental status examination

Glasgow Coma scale Explanation: LOC, a sensitive indicator of neurologic function, is assessed based on the criteria in the Glasgow Coma scale: eye opening, verbal response, and motor response. The Monro-Kellie hypothesis states that because of the limited space for expansion within the skull, an increase in any one of the components (blood, brain tissue, cerebrospinal fluid) causes a change in the volume of the others. Cranial nerve function and the mental status examination would be part of the neurologic examination for this client, but would not be the priority in evaluating LOC.

The nurse practitioner prescribes the medication of choice for an MS patient who is experiencing disabling episodes of muscles spasms, especially at night. Which of the following is the drug most likely prescribed in this scenario? Valium Zanaflex Lioresal Dantrium

Lioresal Explanation: Baclofen (Lioresal), a gamma-aminobutyric acid (GABA) agonist, is the medication of choice for treating spasticity. It can be administered orally or by intrathecal injection.

A nurse is teaching about ischemic stroke prevention to a community group and emphasizes that control of hypertension, which is the major risk factor for stroke, is key to prevention. Ways to control hypertension include the Dietary Approaches to Stop Hypertension (DASH) diet. This diet includes which of the following? Moderate amounts of low-fat dairy products Moderate amounts of animal protein High amounts of low-fat dairy products Moderate amounts of fruits and vegetables

Moderate amounts of low-fat dairy products Explanation: The DASH diet is high in fruits and vegetables, moderate in low-fat dairy products, and low in animal protein.

Which stimulus is known to trigger an episode of autonomic dysreflexia in the client who has suffered a spinal cord injury? Diarrhea Placing the client in a sitting position Placing a blanket over the client Voiding

Placing a blanket over the client Explanation: An object on the skin or skin pressure may precipitate an autonomic dysreflexic episode. In general, constipation or fecal impaction triggers autonomic dysreflexia. When the client is observed to be demonstrating signs of autonomic dysreflexia, the nurse immediately places the client in a sitting position to lower blood pressure. The most common cause of autonomic dysreflexia is a distended bladder.

A client the nurse is caring for experiences a seizure. What would be a priority nursing action? Restrain the client during the seizure. Insert a tongue blade between the teeth. Protect the client from injury. Suction the mouth during the convulsion.

Protect the client from injury. Explanation: The nursing action for a client experiencing a seizure should be to protect the client from being injured. To ensure this, the nurse should turn the client to one side and not restrain client's movements. Inserting a tongue blade between the teeth is not as important as protecting the client from injury. The mouth and the pharynx of the client should be suctioned only after the seizure.

A nurse is performing a neurologic assessment on a client with a stroke and cannot elicit a gag reflex. This deficit is related to cranial nerve (CN) X, the vagus nerve. What will the nurse consider a priority nursing diagnosis? Risk for aspiration Risk for falls Risk for impaired skin integrity Decreased intracranial adaptive capacity

Risk for aspiration

The nurse is caring for a client diagnosed with an ischemic stroke and knows that effective positioning of the client is important. Which of the following should be integrated into the client's plan of care? The client's hip joint should be maintained in a flexed position. The client should be in a supine position unless ambulating. The client should be placed in a prone position for 15 to 30 minutes several times a day. The client should be placed in a Trendelenburg position two to three times daily to promote cerebral perfusion.

The client should be placed in a prone position for 15 to 30 minutes several times a day. Explanation: If possible, the clientis placed in a prone position for 15 to 30 minutes several times a day. A small pillow or a support is placed under the pelvis, extending from the level of the umbilicus to the upper third of the thigh. This helps to promote hyperextension of the hip joints, which is essential for normal gait, and helps prevent knee and hip flexion contractures. The hip joints should not be maintained in flexion and the Trendelenburg position is not indicated.

A patient has an S5 spinal fracture from a fall. What type of assistive device will this patient require? Voice or sip-n-puff controlled electric wheelchair Electric or modified manual wheelchair, needs transfer assistance Cane The patient will be able to ambulate independently.

The patient will be able to ambulate independently. Explanation: Patients with spinal cord injuries from S1 to S5 should be able to ambulate independently, without an assistive device.

A client falls to the floor in a generalized seizure with tonic-clonic movements. Which is the first action taken by the nurse? Insert an airway or bite block. Manually restrain the extremities. Turn client to side-lying position. Monitor vital signs.

Turn client to side-lying position. Explanation: When a client begins to convulse, the highest priority is to maintain airway. This can best be accomplished by turning client to side-lying position, which allows saliva and emesis to drain from the mouth. Turning the client also allows the tongue to fall forward opening the airway. More damage can occur if a bite block is inserted after the seizure has begun. Manually restraining extremities is not recommended. Attempting to take blood pressure is not recommended and pulse rate and respirations during the event will not be beneficial. Monitor vital signs during the postictal phase.

Clinical manifestations of neurogenic shock include which of the following? Select all that apply. Venous pooling in the extremities Bradycardia Warm skin Tachycardia Profuse bilateral sweating

Venous pooling in the extremities Bradycardia Warm skin Explanation: Loss of sympathetic innervation causes a decrease in cardiac output, venous pooling in the extremities, and peripheral vasodilation resulting in mild hypotension, bradycardia, and warm skin. In addition, the patient doe not perspire on the paralyzed portions of the body because sympathetic activity is blocked.

A nurse is teaching a client who has facial muscle weakness and has recently been diagnosed with myasthenia gravis. The nurse should teach the client that myasthenia gravis is caused by: genetic dysfunction. upper and lower motor neuron lesions. decreased conduction of impulses in an upper motor neuron lesion. a lower motor neuron lesion.

a lower motor neuron lesion. Explanation: Myasthenia gravis is characterized by a weakness of muscles, especially in the face and throat, caused by a lower motor neuron lesion at the myoneural junction. It isn't a genetic disorder. A combined upper and lower motor neuron lesion generally occurs as a result of spinal injuries. A lesion involving cranial nerves and their axons in the spinal cord would cause decreased conduction of impulses at an upper motor neuron.

A client with a tentative diagnosis of myasthenia gravis is admitted for a diagnostic workup. Myasthenia gravis is confirmed by: a positive edrophonium (Tensilon) test. Kernig's sign. a positive sweat chloride test. Brudzinski's sign.

a positive edrophonium (Tensilon) test. Explanation: A positive edrophonium test confirms the diagnosis of myasthenia gravis. After edrophonium administration, most clients with myasthenia gravis show markedly improved muscle tone. Kernig's sign and Brudzinski's sign indicate meningitis. The sweat chloride test is used to confirm cystic fibrosis.

A client is experiencing severe pain related to increased ICP. Which analgesic would be ordered for this client to help alleviate pain? codeine hydrocodone morphine fentanyl

codeine Explanation: Avoid administering opioid analgesics, except codeine. Opioids interfere with accurate assessment of neurologic function because they constrict the pupils and depress LOC.

A client is being treated for increased intracranial pressure (ICP). The nurse should ensure that the client does not develop hypothermia because: shivering in hypothermia can increase ICP. hypothermia is indicative of severe meningitis. hypothermia is indicative of malaria. hypothermia can cause death to the client.

shivering in hypothermia can increase ICP. Explanation: Care must be taken to avoid the development of hypothermia because hypothermia causes shivering. Shivering, in turn, can increase intracranial pressure.

When communicating with a client who has sensory (receptive) aphasia, the nurse should: allow time for the client to respond. speak loudly and articulate clearly. give the client a writing pad. use short, simple sentences.

use short, simple sentences. Explanation: Although sensory aphasia allows the client to hear words, it impairs the ability to comprehend their meaning. The nurse should use short, simple sentences to promote comprehension. Allowing time for the client to respond might be helpful but is less important than simplifying the communication. Because the client's hearing isn't affected, speaking loudly isn't necessary. A writing pad is helpful for clients with expressive, not receptive, aphasia.

Thrombolytic therapy should be initiated within what time frame of an ischemic stroke to achieve the best functional outcome? 3 hours 6 hours 9 hours 12 hours

3 hours Explanation: Rapid diagnosis of stroke and initiation of thrombolytic therapy (within 3 hours) in clients with ischemic stroke lead to a decrease in the size of the stroke and an overall improvement in functional outcome after 3 months.

A client is brought to the emergency department with symptoms of a cerebrovascular accident (CVA). The nurse would anticipate which diagnostic evaluation to be completed prior to initiation of treatment? Prothrombin level Chest x-ray Brain CT scan or MRI Lumbar puncture

Brain CT scan or MRI Explanation: CT scan or MRI differentiates CVA from other disorders and can differentiate between ischemic or hemorrhagic strokes. PT level would be done if the client is receiving anticoagulant therapy. Chest x-ray may be performed if respiratory concerns are indicated. Lumbar puncture would be done if subarachnoid bleeding is suspected.

The nurse is expecting to admit a client with a diagnosis of meningitis. While preparing the client's room, which of the following would the nurse most likely have available? Equipment to maintain infection control precautions Nasogastric tubing Extra lighting IV tensilon

Equipment to maintain infection control precautions Explanation: An important component of nursing care for the client with meningits is instituting infection control precautions until 24 hours after initiation of antibiotic therapy. Oral and nasal discharge is considered infectious. This client may well experience photophobia, so the lighting should be kept dim. IV Tensilon is used to diagnose myasthenia gravis.

A client is brought by ambulance to the ED after suffering what the family thinks is a stroke. The nurse caring for this client is aware that an absolute contraindication for thrombolytic therapy is what? Evidence of hemorrhagic stroke Blood pressure of ≥ 180/110 mm Hg Evidence of stroke evolution Previous thrombolytic therapy within the past 12 months

Evidence of hemorrhagic stroke Explanation: Thrombolytic therapy would exacerbate a hemorrhagic stroke with potentially fatal consequences. Stroke evolution, high BP, or previous thrombolytic therapy does not contraindicate its safe and effective use.

Which of the following are disease-modifying agents used in the treatment of multiple sclerosis (MS)? Select all that apply. Interferon beta-1a (Rebif) Interferon beta-1b (Betaseron) Interferon beta-1a (Avonex) Glatiramer acetate (Copaxone) Tizanidine (Zanaflex)

Interferon beta-1a (Rebif) Interferon beta-1b (Betaseron) Interferon beta-1a (Avonex) Glatiramer acetate (Copaxone) Explanation: Disease-modifying agents include Rebif, Betaseron, Avonex, and Copaxone. Zanaflex is a benzodiazepine used to treat spasticity in the treatment of MS.

A patient with epilepsy is having a seizure. Which of the following should the nurse do after the seizure? Keep the patient to one side. Place a cooling blanket beneath the patient. Help the patient sit up. Pry the patient's mouth open to allow a patent airway.

Keep the patient to one side.

A nurse is working with a student nurse who is caring for a client with an acute bleeding cerebral aneurysm. Which action by the student nurse requires further intervention? Positioning the client to prevent airway obstruction Keeping the client in one position to decrease bleeding Administering I.V. fluid as ordered and monitoring the client for signs of fluid volume excess Maintaining the client in a quiet environment

Keeping the client in one position to decrease bleeding Explanation: The student nurse shouldn't keep the client in one position. She should carefully reposition the client often (at least every hour). The client needs to be positioned so that a patent airway can be maintained. Fluid administration must be closely monitored to prevent complications such as increased intracranial pressure. The client must be maintained in a quiet environment to decrease the risk of rebleeding.

Which of the following is a clinical manifestation of pupillary changes that indicate increasing ICP? Pupils are equal and normally reactive. Pupils are unequal in diameter. Pupils are showing progressive dilation. Pupils are fixed and dilated.

Pupils are showing progressive dilation. Explanation: Pupils that show progressive dilation are indicative of increasing intracranial pressure. Refer to Table 45-1 in the text.

A male patient with cerebrovascular accident (CVA) is prescribed medication to treat the disorder. The patient wants to know what other measures may help reduce CVA. Which of the following is an accurate suggestion for the patient? Reduce hypertension and high blood cholesterol levels. Increase body weight moderately. Increase the intake of proteins and carbohydrates. Increase the fluids and hydration.

Reduce hypertension and high blood cholesterol levels. Explanation: CVAs are prevented by reducing certain risk factors, such as hypertension, overweight, cardiac dysrhythmias (such as atrial fibrillation), and high blood cholesterol levels. Patients should not gain body weight. In addition, the increased intake of proteins, carbohydrates, or fluids does not help in reducing the risk of CVAs.

A client has tension headaches. The nurse recommends massage as a treatment for tension headaches. How does massage help clients with tension headaches? Reduces hypotension Increases appetite Relaxes muscles Relieves migraines

Relaxes muscles Explanation: Massaging relaxes tense muscles, causes local dilation of blood vessels, and relieves headache. However, this approach is not likely to help a client with migraine or cluster headaches. Massage is not offered to clients with tension headaches to increase their appetite or reduce hypotension.

A nurse is caring for a client with a history of severe migraines. The client has a medical history that includes asthma, gastroesophageal reflux disease, and three pregnancies. Which medication does the nurse anticipate the physician will order for the client's migraines? Verapamil (Calan) Metoprolol (Lopressor) Amiodarone (Cordarone) Captopril (Coreg)

Verapamil (Calan) Explanation: Calcium channel blockers, such as verapamil, and beta-adrenergic blockers, such as metoprolol, are commonly used to treat migraines because they help control cerebral blood vessel dilation. Calcium channels blockers, however, are ordered for clients who may not be able to tolerate beta-adrenergic blockers, such as those with asthma. Amiodarone and captopril aren't used to treat migraines.

In which specific instances should the nurse assess the client's cranial nerves? Select all that apply. When a neurogenic bladder develops When level of consciousness is decreased With brain stem pathology In the presence of peripheral nervous system disease When a spinal reflex is interrupted

When level of consciousness is decreased With brain stem pathology In the presence of peripheral nervous system disease Explanation: Cranial nerves are assessed when level of consciousness is decreased, with brain stem pathology, or in the presence of peripheral nervous system disease. Abnormalities in muscle tone and involuntary movements are less likely to prompt the assessment of cranial nerves, since these nerves do not directly mediate most aspects of muscle tone and movement.

A nurse is teaching a client with multiple sclerosis (MS). When teaching the client how to reduce fatigue, the nurse should tell the client to: take a hot bath. rest in an air-conditioned room. increase the dose of muscle relaxants. avoid naps during the day.

rest in an air-conditioned room. Explanation: Fatigue is a common symptom in clients with MS. Lowering the body temperature by resting in an air-conditioned room may relieve fatigue; however, extreme cold should be avoided. A hot bath or shower can increase body temperature, producing fatigue. Muscle relaxants, ordered to reduce spasticity, can cause drowsiness and fatigue. Frequent rest periods and naps can relieve fatigue. Other measures to reduce fatigue in the client with MS include treating depression, using occupational therapy to learn energy-conservation techniques, and reducing spasticity.

Aneurysm rebleeding occurs most frequently during which time frame after the initial hemorrhage? First 2 weeks First 48 hours First week First month

First 2 weeks Explanation: Aneurysm rebleeding occurs most frequently during the first 2 weeks after the initial hemorrhage and is considered a major complication.

Which is the most common cause of acute encephalitis in the United States? Western equine bacteria Herpes simplex virus (HSV) Lyme Disease Human immunodeficiency virus (HIV)

Herpes simplex virus (HSV) Explanation: HSV-1 ( herpes simplex virus) is the most common cause of acute encephalitis in the United States. Fungal infections of the central nervous system occur rarely in healthy people. The Western equine encephalitis virus is one of four types of arboviral encephalitis that occur in North America is one of several fungi that may cause fungal encephalitis. Lyme disease leads to flu like symptoms and starts as a local infection which can systematically spread causing organ issues, however the incidence is rate, HIV leads to autoimmune disorders.

A client with a cerebrovascular accident (CVA) is having difficulty with eating food on the plate. Which is the best priority nursing action to be taken? Perform a vision field assessment. Reposition the tray and plate. Assist the client with feeding. Know this is a normal finding for CVA.

Perform a vision field assessment. Explanation: The nurse should perform a vision field assessment to evaluate the client forhemianopia. This finding could indicate damage to the visual area of the brain as a result of evolving CVA. Repositioning the tray and assisting with feeding would not be the best nursing action until new finding has been evaluated. Hemianopia can be associated with a CVA but, when presenting as a new finding, should be evaluated and reported immediately.

After striking his head on a tree while falling from a ladder, a client is admitted to the emergency department. He's unconscious and his pupils are nonreactive. Which intervention should the nurse question? Giving him a barbiturate Placing him on mechanical ventilation Performing a lumbar puncture Elevating the head of his bed

Performing a lumbar puncture Explanation: The client's history and assessment suggest that he may have increased intracranial pressure (ICP). If this is the case, lumbar puncture shouldn't be done because it can quickly decompress the central nervous system, causing additional damage. After a head injury, barbiturates may be given to prevent seizures; mechanical ventilation may be required if breathing deteriorates; and elevating the head of the bed may be used to reduce ICP.

The nurse is caring for a client experiencing autonomic dysreflexia. Which of the following does the nurse recognize as the source of symptoms? Autonomic nervous system Central nervous system Peripheral nervous system Sympathetic nervous system

Sympathetic nervous system Explanation: The nurse recognizes that autonomic dysreflexia is an exaggerated sympathetic nervous system response. Symptoms include severe hypertension, slow heart rate, pounding headache, etc. and can lead to seizures, stroke, and death. The autonomic nervous system regulates "feed and breed" functions. The central and peripheral nervous system is a component of the sympathetic nervous system.

A nurse is caring for a client with recent history of migraines. What aspect of this client's current status may rule out the safe use of triptans? The client is 75 years old. The client's migraines are linked to psychosocial stress. The client has angina. The client has hypertension.

The client has angina. Explanation: Triptans can cause chest pain and are contraindicated in clients with ischemic heart disease. Hypertension, older age, and the presence of stress do not rule out the use of triptans.

A client recovering from a stroke has severe shoulder pain from subluxation of the shoulder. To prevent further injury and pain, the nurse caring for this client is aware of what principle of care? The client should be fitted with a cast because use of a sling should be avoided due to adduction of the affected shoulder. Elevation of the arm and hand can lead to further complications associated with edema. Passively exercising the affected extremity is avoided in order to minimize pain. The client should be taught to interlace fingers, place palms together, and slowly bring scapulae forward to avoid excessive force to shoulder.

The client should be taught to interlace fingers, place palms together, and slowly bring scapulae forward to avoid excessive force to shoulder. Explanation: To prevent shoulder pain, the nurse should never lift a client by the flaccid shoulder or pull on the affected arm or shoulder. The client is taught how to move and exercise the affected arm/shoulder through proper movement and positioning. The client is instructed to interlace the fingers, place the palms together, and push the clasped hands slowly forward to bring the scapulae forward; he or she then raises both hands above the head. This is repeated throughout the day. The use of a properly worn sling when the client is out of bed prevents the paralyzed upper extremity from dangling without support. Range-of-motion exercises are still vitally important in preventing a frozen shoulder and ultimately atrophy of subcutaneous tissues, which can cause more pain. Elevation of the arm and hand is also important in preventing dependent edema of the hand.

A nurse is preparing a client for a computed tomography (CT) scan that requires infusion of radiopaque dye. Which question is the most important for the nurse to ask? "When did you last have something to eat or drink?" "When did you last take any medication?" "Are you allergic to seafood or iodine?" "How much do you weigh?"

"Are you allergic to seafood or iodine?" Seafood and the radiopaque dye used in CT contain iodine. To prevent an allergic reaction to the radiopaque dye, the nurse should ask the client about allergies to seafood or iodine before the CT scan. Because fasting is unnecessary before a CT scan, the nurse doesn't need to obtain information about the client's last food and fluid intake. The client's last dose of medication and current weight also are irrelevant.

A nurse practitioner is presenting health information about strokes at a clinic. She mentions that there are five categories of strokes based on their origin. Which of the following is the category that has the highest incidence of strokes (30%)? Cardiogenic embolic Cryptogenic Large artery thrombotic Small artery thrombotic

Cryptogenic Explanation: Cryptogenic strokes, which account for 30% of all strokes, refer to strokes that cannot be attributed to any specific cause.

The statements presented here match nursing interventions with nursing diagnoses. Which statements are appropriate for a client who has suffered a head injury? Select all that apply. Ineffective airway clearance: Apply suction as indicated Deficient fluid volume: Administer 1 L of normal saline daily Disturbed sleep pattern: Provide back rubs to the client Ineffective cerebral tissue perfusion: Maintain cerebral perfusion pressure ≤50 mm Hg Interrupted family process: Encourage the family to join a support group

Ineffective airway clearance: Apply suction as indicated Disturbed sleep pattern: Provide back rubs to the client Interrupted family process: Encourage the family to join a support group Explanation: These nursing diagnoses match the interventions correctly. The goal of hydration is to prevent dehydration or fluid overload; fluid replacement is based on the client's individual needs. Cerebral perfusion pressure should be maintained between 50 and 70 mm Hg.

A client with meningitis has a history of seizures. Which activity should the nurse do while the client is actively seizing? Place a cooling blanket beneath the client Provide oxygen or anticonvulsants, whichever is available Turn the client to the side during a seizure and do not restrain movements Suction the client's mouth and pharynx

Turn the client to the side during a seizure and do not restrain movements Explanation: When a client is in a seizure, the nurse should turn the client to the side and not restrain his or her movements. This helps reduce the potential for aspiration of saliva or stomach contents. The nurse should suction the mouth and pharynx after a seizure has occurred, not during the seizure. Anticonvulsants may be administered to reduce the chances of seizure. Oxygen should not be given to clients with seizures. Clients with respiratory distress are given oxygen. Finally, a cooling blanket is placed beneath the client when hyperthermia occurs, not a seizure.

A client suffered a closed head injury in a motor vehicle collision, and an ICP monitor was inserted. In the occurrence of increased ICP, what physiologic function contributes to the increase in intracranial pressure? vasodilation vasoconstriction hypertension increased PaO

vasodilation Explanation: Hypotension and hypoxia lead to vasodilation, which contributes to increased ICP, compressing blood vessels and leading to cerebral ischemia. As ICP continues to rise, autoregulatory mechanisms can become compromised; hypotension and hypoxia lead to vasodilation, which contributes to increased ICP.

A nurse conducts the Romberg test on a client by asking the client to stand with the feet close together and the eyes closed. As a result of this posture, the client suddenly sways to one side and is about to fall when the nurse intervenes and saves the client from being injured. How should the nurse interpret the client's result? Positive Romberg test, indicating a problem with level of consciousness Negative Romberg test, indicating a problem with body mass Negative Romberg test, indicating a problem with vision Positive Romberg test, indicating a problem with equilibrium

Positive Romberg test, indicating a problem with equilibrium If the client sways and starts to fall during the Romberg test, it indicates a positive result. This means the client has a problem with equilibrium. The examiner or the nurse stands fairly close to the client during the test to prevent the client from falling. The Romberg test is used to assess the client's motor function, including muscle movement, size, tone, strength, and coordination. However, the Romberg test is not used to assess the client's level of consciousness, body mass, or vision.

An elderly woman found with a head injury on the floor of her home is subsequently admitted to the neurologic ICU. What is the best rationale for the following health care provider prescriptions: elevate the HOB; keep the head in neutral alignment with no neck flexion or head rotation; avoid sharp hip flexion? To decrease cerebral arterial pressure To avoid impeding venous outflow To prevent flexion contractures To prevent aspiration of stomach contents

To avoid impeding venous outflow Explanation: Any activity or position that impedes venous outflow from the head may contribute to increased volume inside the skull and possibly increase ICP. Cerebral arterial pressure will be affected by the balance between oxygen and carbon dioxide. Flexion contractures are not a priority at this time. Stomach contents could still be aspirated in this position.

In planning care for a patient with an extrapyramidal disorder, the nurse recognizes that a major difference between Parkinson's disease and Huntington's disease is the development of ________ in clients with advanced Huntington's disease. hallucinations and delusions depression bradykinesia muscle fasciculations

hallucinations and delusions Explanation: As Huntington's disease progresses, hallucinations, delusions, and impaired judgment develop due to degeneration of the cerebral cortex. Depression is a likely symptom for clients with both Parkinson's disease and Huntington's disease. Bradykinesia, slowness in performing spontaneous movement, is commonly associated with Parkinson's disease. Muscle fasciculations, or twitching, are commonly associated with ALS.

A client is admitted to the neurologic ICU with a spinal cord injury. When assessing the client the nurse notes there is a sudden depression of reflex activity in the spinal cord below the level of injury. What should the nurse suspect? Epidural hemorrhage Hypertensive emergency Spinal shock Hypovolemia

Spinal shock Explanation: In spinal shock, the reflexes are absent, BP and heart rate fall, and respiratory failure can occur. Hypovolemia, hemorrhage, and hypertension do not cause this sudden change in neurologic function.

A client reports light-headedness, speech disturbance, and left-sided weakness lasting for several hours. The neurologist diagnosed a transient ischemic attack, which caused the client great concern. What would the nurse include during client education? When symptoms cease, the client will return to presymptomatic state. A TIA is an insidious, often chronic episode of neurologic impairment. Symptoms of a TIA may linger for up to a week. Two thirds of people that experience a TIA will go on to develop a stroke.

When symptoms cease, the client will return to presymptomatic state. Explanation: Impaired blood circulation can be caused by arteriosclerosis, cardiac disease, or diabetes. A TIA is a sudden, brief episode of neurologic impairment. Symptoms may disappear within 1 hour; some continue for as long as 1 day. One third of people who experience a TIA subsequently develop a stroke.

A client is scheduled for an EEG after having a seizure for the first time. Client preparation for this test should include which instruction? "Don't eat anything for 12 hours before the test." "Don't shampoo your hair for 24 hours before the test." "Avoid stimulants and alcohol for 24 to 48 hours before the test." "Avoid thinking about personal matters for 12 hours before the test."

"Avoid stimulants and alcohol for 24 to 48 hours before the test." Explanation: For 24 to 48 hours before an EEG, the client should avoid coffee, cola, tea, alcohol, and cigarettes because these may interfere with the accuracy of test results. (For the same reason, the client also should avoid antidepressants, sedatives, and anticonvulsants.) To avoid a reduced serum glucose level, which may alter test results, the client should eat normal meals before the test. The hair should be washed before an EEG because the electrodes must be applied to a clean scalp. The client's thoughts don't affect the test results.

A rehabilitation nurse caring for a client who has had a stroke is approached by the client's family and asked why the client has to do so much for herself when she is obviously struggling. What would be the nurse's best answer? "We are trying to help her be as useful as she possibly can." "The focus on care in a rehabilitation facility is to help the client to resume as much self-care as possible." "We aren't here to care for her the way the hospital staff did; we are here to help her get better so she can go home." "Rehabilitation means helping clients do exactly what they did before their stroke."

"The focus on care in a rehabilitation facility is to help the client to resume as much self-care as possible." Explanation: In both acute care and rehabilitation facilities, the focus is on teaching the client to resume as much self-care as possible. The goal of rehabilitation is not to be "useful," nor is it to return clients to their prestroke level of functioning, which may be unrealistic.

A nurse is caring for a client with dementia. A family member of the client asks what the most common cause of dementia is. Which response by the nurse is most appropriate? "Dementia is a terrible disease of the elderly." "The most common cause of dementia in the elderly is Alzheimer's disease." "Drug interactions are the most common cause of dementia in the elderly." "Depression may manifest as dementia in elderly clients."

"The most common cause of dementia in the elderly is Alzheimer's disease." Explanation: The nurse should inform the family member that Alzheimer's disease is the most common cause of dementia in elderly clients. Dementia is a clinical manifestation, not a disease process. Although drug interactions and overmedication are causes of dementia, these causes aren't as common as Alzheimer's disease. Depression is common in elderly clients, but it doesn't cause dementia.

A nurse is teaching a client who was recently diagnosed with myasthenia gravis. Which statement should the nurse include in her teaching? "You'll continue to experience progressive muscle weakness and sensory deficits." "You'll need to take edrophonium (Tensilon) to treat the disease." "The disease is a disorder of motor and sensory dysfunction." "This disease doesn't cause sensory impairment."

"This disease doesn't cause sensory impairment." Explanation: Myasthenia gravis affects motor function; therefore, the nurse should inform the client that sensory impairments won't occur. This disease is chronic; there's no cure. It can be managed with edrophonium in the diagnostic phase; however, this drug isn't used to treat the condition.

A client with Parkinson's disease asks the nurse what their treatment is supposed to do since the disease is progressive. What would be the nurse's best response? "Treatment aims at keeping you independent as long as possible." "Treatment really doesn't matter; the disease is going to progress anyway." "Treatment for Parkinson's is only palliative; it keeps you comfortable." "Treatment aims at keeping you emotionally healthy by making you think you are doing something to fight this disease."

"Treatment aims at keeping you independent as long as possible." Explanation: Treatment aims at prolonging independence. Treatment does matter, it is not palliative, and it is not aimed at keeping you emotionally healthy.

The nurse is caring for a patient in the neurologic ICU who sustained head trauma in a physical altercation. What would the nurse know is an optimal range of ICP for this patient? 8 to 15 mm Hg 0 to 10 mm Hg 20 to 30 mm Hg 25 to 40 mm Hg

0 to 10 mm Hg Explanation: ICP is usually measured in the lateral ventricles, with the normal pressure being 0 to 10 mm Hg, and 15 mm Hg being the upper limit of normal (Hickey, 2009).

While stopped at a stop sign, a patient's car was struck from behind by another vehicle. The patient sustained a cerebral contusion and was admitted to the hospital. During what time period after the injury will the effects of injury peak? 6 to 8 hours 18 to 36 hours 12 to 24 hours 48 to 72 hours

18 to 36 hours Explanation: Contusions are characterized by loss of consciousness associated with stupor and confusion. Other characteristics can include tissue alteration and neurologic deficit without hematoma formation, alteration in consciousness without localizing signs, and hemorrhage into the tissue that varies in size and is surrounded by edema. The effects of injury (hemorrhage and edema) peak after about 18 to 36 hours.

When planning care for a client with a head injury, which position should the nurse include in the care plan to enhance client outcomes? Trendelenburg's 30-degree head elevation Flat Side-lying

30-degree head elevation Explanation: For clients with increased intracranial pressure (ICP), the head of the bed should be elevated to 30 degrees to promote venous outflow. Trendelenburg's position is contraindicated because it can raise ICP. Flat or neutral positioning is indicated when elevating the head of the bed would increase the risk of neck injury or airway obstruction. A side-lying position isn't specifically a therapeutic treatment for increased ICP.

The earliest sign of serious impairment of brain circulation related to increased ICP is: A bounding pulse. Bradycardia. Hypertension. A change in consciousness.

A change in consciousness. Explanation: The earliest sign of increasing ICP is a change in the LOC. Any changes in LOC should be reported immediately.

The nurse is caring for a client who was involved in a motorcycle accident 7 days ago. Since admission the client has been unresponsive to painful stimuli. The client had a ventriculostomy placed upon admission to the ICU. The current assessment findings include ICP of 14 with good waveforms, pulse 92, respirations per ventilator, temperature 102.7°F (rectal), urine output 320 mL in 4 hours, pupils pinpoint and briskly reactive, and hot, dry skin. Which is the priority nursing action? Inspect the ICP monitor to ensure it is working properly. Administer acetaminophen per orders. Provide ventriculostomy care. Assess for signs and symptoms of infection.

Administer acetaminophen per orders. Explanation: The nurse needs to control the fever by administering the ordered acetaminophen as the priority action. An increase in the client's temperature can lead to increased cerebral metabolic demands and poor outcomes if not properly treated. The nurse should always inspect the equipment to ensure that it is working properly, but this is not the priority because there is no indication of equipment failure. The nurse should provide ventriculostomy care, but this is not the priority as there is an elevated temperature. Because the client has an elevated temperature, the nurse should assess for signs and symptoms of infection, but only after treating the elevated temperature.

The nurse is caring for a male client who has emerged from a coma following a head injury. The client is agitated. Which intervention will the nurse implement to prevent injury to the client? Administer opioids to control restlessness Apply an external urinary sheath catheter Provide a dimly lit room to prevent visual hallucinations Turn and reposition the client every 2 hours

Apply an external urinary sheath catheter Explanation: A strategy the nurse can implement to prevent client injury is to use an external sheath catheter for a male client if incontinence occurs. Because prolonged use of an indwelling catheter inevitably produces infection, the client may be placed on an intermittent catheterization schedule. Opioids are contraindicated because they depress respirations, constrict the pupils, and alter responsiveness. Providing adequate lighting to prevent visual hallucinations is recommended. Repositioning the client every 2 hours maintains skin integrity.

A client is admitted for scheduled gamma-knife radiosurgery, in the treatment of a brain tumor. Which nursing measure is primary in the postsurgical care of this client? Assessing skull dressing for excess drainage Time, distance, and shielding against radiation Assess neurological findings. Maintain airway via artificial ventilation.

Assess neurological findings. Explanation: Gamma-knife radiosurgery is a non-invasive alternative for treating tumors within the brain. The nurse would be responsible for completing a neurological assessment on the client and providing comfort measures as needed. There is no incision on the skull, and no risk for radiation exposure to the nurse. The procedure eliminates surgical and anesthesia complications and does not result in use of a ventilator or artificial airway maintenance.

A client in the emergency department has bruising over the mastoid bone and rhinorrhea. The triage nurse suspects the client has which type of skull fracture? Linear Simple Basilar Comminuted

Basilar Explanation: An area of ecchymosis (bruising) may be seen over the mastoid (Battle sign) in a basilar skull fracture. Basilar skull fractures are also suspected when cerebrospinal fluid (CSF) escapes from the ears (CSF otorrhea) and the nose (CSF rhinorrhea). A simple (linear) fracture is a break in bone continuity. A comminuted fracture refers to a splintered or multiple fracture line.

A client who just experienced a suspected ischemic stroke is brought to the ED by ambulance. On what should the nurse's primary assessment focus? Cardiac and respiratory status Seizure activity Pain Fluid and electrolyte balance

Cardiac and respiratory status Explanation: Acute care begins with managing ABCs. Clients may have difficulty keeping an open and clear airway secondary to decreased LOC. Neurologic assessment with close monitoring for signs of increased neurologic deficit and seizure activity occurs next. Fluid and electrolyte balance must be controlled carefully with the goal of adequate hydration to promote perfusion and decrease further brain activity.

The causes of acquired seizures include what? (Mark all that apply.) Cerebrovascular disease Metabolic and toxic conditions Hypernatremia Brain tumor Drug and alcohol withdrawal

Cerebrovascular disease Metabolic and toxic conditions Brain tumor Drug and alcohol withdrawal Explanation: The specific causes of seizures are varied and can be categorized as idiopathic (genetic, developmental defects) and acquired. Causes of acquired seizures include cerebrovascular disease; hypoxemia of any cause, including vascular insufficiency; fever (childhood); head injury; hypertension; central nervous system infections; metabolic and toxic conditions (eg, renal failure, hyponatremia, hypocalcemia, hypoglycemia, pesticide exposure); brain tumor; drug and alcohol withdrawal; and allergies.

A nurse is providing care to a client with Parkinson's disease. The nurse understands the the client's signs and symptoms are related to a depletion of which of the following? Serotonin Acetylcholine Dopamine Norepinephrine

Dopamine Explanation: Parkinson's disease is associated with decreased levels of dopamine resulting from destruction of pigmented neuronal cells in the substantia nigra in the basal ganglion region. The loss of dopamine stores results in more excitatory neurotransmitters (acetylcholine) than inhibitory transmitters (dopamine). Serotonin and norepinephrine are not involved.

When caring for a client with trigeminal neuralgia, which intervention has the highest priority? Providing emotional support while the client adjusts to changes in his physical appearance Monitoring intake and output Assisting with ambulation Encouraging the client to bathe with care

Encouraging the client to bathe with care Explanation: Trigeminal neuralgia is a common disorder that causes severe pain along the trigeminal nerve (the nerve affecting the face) and surrounding areas. The nurse should encourage to the client to be gentle when bathing because performing facial hygiene can cause pain. The client's facial appearance doesn't change, so it isn't necessary to provide emotional support for changes in physical appearance. The nurse doesn't need to make monitoring intake and output and assisting with ambulation priorities because these parameters aren't affected by disorders of the trigeminal nerve.

Which nursing interventions might need to be considered in a care plan for a client with advanced multiple sclerosis? Select all that apply. Ensure access to a language board when communicating with the client. Obtain daily weights to monitor weight gain. Establish a voiding time schedule. Encourage the client to walk with feet wide apart.

Ensure access to a language board when communicating with the client. Establish a voiding time schedule. Encourage the client to walk with feet wide apart. Explanation: Language assistive devices may be needed if communication is severely affected. Occasional bladder incontinence may lead to total incontinence. A voiding time schedule will allow the client greater independence. If motor dysfunction causes problems of incoordination and clumsiness, the patient is at risk for falling. As the disease progresses, nutritional deficiencies may develop. Weight should be assessed to ensure that there is no significant weight loss.

Which of the following symptoms are indicative of a rapidly expanding acute subdural hematoma? Select all that apply. Hemiparesis Tachypnea Decreased reactivity of the pupils Bradycardia Hypotension Coma

Hemiparesis Decreased reactivity of the pupils Bradycardia Coma Explanation: Signs and symptoms include changes in the level of consciousness (LOC), changes in the reactivity of the pupils, and hemiparesis (weakness on one side of the body). There may be minor or even no symptoms, with small collections of blood. Coma, increasing blood pressure, decreasing heart rate, and slowing respiratory rate are all signs of a rapidly expanding mass requiring immediate intervention.

A client undergoes cerebral angiography for evaluation after an intracranial computed tomography scan revealed a subarachnoid hemorrhage. Afterward, the nurse checks frequently for signs and symptoms of complications associated with this procedure. Which findings indicate spasm or occlusion of a cerebral vessel by a clot? Nausea, vomiting, and profuse sweating Hemiplegia, seizures, and decreased level of consciousness (LOC) Difficulty breathing or swallowing Tachycardia, tachypnea, and hypotension

Hemiplegia, seizures, and decreased level of consciousness (LOC) Explanation: Spasm or occlusion of a cerebral vessel by a clot causes signs and symptoms similar to those of a stroke: hemiplegia, seizures, decreased LOC, aphasia, hemiparesis, and increased focal symptoms. Nausea, vomiting, and profuse sweating suggest a delayed reaction to the contrast medium used in cerebral angiography. Difficulty breathing or swallowing may signal a hematoma in the neck. Tachycardia, tachypnea, and hypotension suggest internal hemorrhage.

A critical care nurse is caring for a client with a cerebrovascular accident (CVA) The client is prescribed heparin for treatment. The nurse knows to monitor the client for what adverse effects? Migraine attacks Hemorrhage Respiratory distress High blood pressure

Hemorrhage Explanation: A client with a CVA who is given heparin should be monitored for hemorrhage and bleeding at the subcutaneous injection site. Respiratory distress, high blood pressure, or migraine attacks are not likely to occur in such a client.

A client is receiving an IV infusion of mannitol (Osmitrol) after undergoing intracranial surgery to remove a brain tumor. To determine whether this drug is producing its therapeutic effect, the nurse should consider which finding most significant? Decreased level of consciousness (LOC) Elevated blood pressure Increased urine output Decreased heart rate

Increased urine output Explanation: The therapeutic effect of mannitol is diuresis, which is confirmed by an increased urine output. A decreased LOC and elevated blood pressure may indicate lack of therapeutic effectiveness. A decreased heart rate doesn't indicate that mannitol is effective.

The nurse is caring for a client with Guillain-Barré syndrome (GBS). The client also has an ascending paralysis. Knowing the potential complications of the disorder, what should the nurse keep always ready at the bedside? Nebulizer and thermometer Intubation tray and suction apparatus Blood pressure apparatus Incentive spirometer

Intubation tray and suction apparatus Explanation: Progressive GBS can move to the upper areas of the body and affect the muscles of respiration. If the respiratory muscles are involved, endotracheal intubation and mechanical ventilation become necessary. A spirometer is used to evaluate the client's ventilation capacity. A blood pressure apparatus, nebulizer, and thermometer are not required because generally a client with GBS does not show signs of increased blood pressure or temperature.

During a client's recovery from stroke, the nurse should be aware of predictors of stroke outcome in order to help clients and families set realistic goals. What are the predictors of stroke outcome? Select all that apply. National Institutes of Health Stroke Scale (NIHSS) score Race LOC at time of admission Gender Age

National Institutes of Health Stroke Scale (NIHSS) score LOC at time of admission Age Explanation: It is helpful for clinicians to be knowledgeable about the relative importance of predictors of stroke outcome (age, NIHSS score, and LOC at time of admission) to provide stroke survivors and their families with realistic goals. Race and gender are not significant predictors of stroke outcome.

The nurse is caring for a client who was discovered unconscious after falling off a ladder. The client is diagnosed with a concussion. All testing is normal, and discharge instructions are compiled. Which instructions have been compiled for the spouse? Tylenol may be administered for aches. Observe for any signs of behavioral changes. A light meal may be eaten if desired. Follow up with regular physician is encouraged.

Observe for any signs of behavioral changes. Explanation: All of the options are typical for a client being discharged with a concussion. The instruction that is emphasized is to observe for any signs of behavior changes, which may indicate an increase in the client's intracranial pressure. A concussion results in diffuse or microscopic injury to the brain with symptoms that may evolve.

A client has had an ischemic stroke and has been admitted to the medical unit. What action should the nurse perform to best prevent joint deformities? Place the client in the prone position for 30 minutes/day. Assist the client in acutely flexing the thigh to promote movement. Place a pillow in the axilla when there is limited external rotation. Place the client's hand in pronation.

Place a pillow in the axilla when there is limited external rotation. Explanation: A pillow in the axilla prevents adduction of the affected shoulder and keeps the arm away from the chest. The prone position with a pillow under the pelvis, not flat, promotes hyperextension of the hip joints, essential for normal gait. To promote venous return and prevent edema, the upper thigh should not be flexed acutely. The hand is placed in slight supination, not pronation, which is its most functional position.

The nurse is caring for a client immediately after a spinal cord injury. Which assessment finding is essential when caring for a client in spinal shock with injury in the lower thoracic region? Numbness and tingling Respiratory pattern Pulse and blood pressure Pain level

Pulse and blood pressure Explanation: Spinal shock is a loss of sympathetic reflex activity below the level of the injury within 30 to 60 minutes after insult. In addition to the paralysis, manifestations include pronounced hypotension, bradycardia, and warm, dry skin. Numbness and tingling and pain are not as high of a concern at this time due to the cord injury. Because the level of impairment is below the first thoracic vertebrae, respiratory failure is not a concern.

As a member of the stroke team, the nurse knows that thrombolytic therapy carries the potential for benefit and for harm. The nurse should be cognizant of what contraindications for thrombolytic therapy? Select all that apply. INR above 1.0 Recent intracranial pathology Sudden symptom onset Current anticoagulation therapy Symptom onset greater than 3 hours prior to admission

Recent intracranial pathology Current anticoagulation therapy Symptom onset greater than 3 hours prior to admission Explanation: Some of the absolute contraindications for thrombolytic therapy include symptom onset greater than 3 hours before admission, a client who is anticoagulated (with an INR above 1.7), or a client who has recently had any type of intracranial pathology (e.g., previous stroke, head injury, trauma).

A client has an exacerbation of multiple sclerosis. The physician orders dantrolene (Dantrium), 25 mg P.O. daily. Which assessment finding indicates the medication is effective? Increased ability to sleep Relief from constipation Relief from pain Reduced muscle spasticity

Reduced muscle spasticity Explanation: Dantrolene reduces muscle spacticity. It doesn't increase the ability to sleep or relieve constipation or pain.

A client diagnosed with Huntington's disease has developed severe depression. What would be most important for the nurse to assess for? Loss of bowel and bladder control Choreiform movements Suicidal ideations Emotional apathy

Suicidal ideations Explanation: Severe depression is common and can lead to suicide, so it is most important for the nurse to assess for suicidal ideations. Symptoms of Huntington's disease develop slowly and include mental apathy and emotional disturbances, choreiform movements (uncontrollable writhing and twisting of the body), grimacing, difficulty chewing and swallowing, speech difficulty, intellectual decline, and loss of bowel and bladder control. Assessing for these symptoms is appropriate but not as important as assessing for suicidal ideations.

Vagus nerve demyelinization, which may occur in Guillain-Barré syndrome, is manifested by which of the following? Tachycardia Bulbar weakness Blindness Inability to swallow

Tachycardia Explanation: Cranial nerve demyelination can result in a variety of clinical manifestations. Optic nerve demyelination may result in blindness. Bulbar muscle weakness related to demyelination of the glossopharyngeal and vagus nerves results in the inability to swallow or clear secretions. Vagus nerve demyelination results in autonomic dysfunction, manifested by instability of the cardiovascular system. The presentation is variable and may include tachycardia, bradycardia, hypertension, or orthostatic hypotension.

When developing a care plan for a client who has recently suffered a stroke, a nurse includes the nursing diagnosis Risk for imbalanced body temperature. What is the rationale for this diagnosis? The stroke may have impacted the body's thermoregulation centers. A decreased body temperature will signal the need to cover the client. An elevated temperature indicates cerebellum malfunction. An elevated body temperature indicates infection.

The stroke may have impacted the body's thermoregulation centers. Explanation: The body's thermoregulation centers are located in the hypothalamus. A stroke may impair their functioning. A decreased body temperature isn't necessarily an indication to cover the client. Although an elevated temperature may indicate cerebellum malfunction or infection, these factors aren't the focus of the risk described in the nursing diagnosis.

The client presents to the walk-in clinic with fever, nuchal rigidity, and headache. Which of the following assessment findings would be most significant in the diagnosis of this client? Change in level of consciousness Vomiting Vector bites Seizures

Vector bites Explanation: Possible exposure to mosquito bites can be beneficial in the diagnosing of encephalitis secondary to West Nile virus. Change in level of consciousness (LOC), vomiting, and seizures are all symptoms of increased intracranial pressure (ICP) and do not assist in the differentiating of cause, diagnosis, or establishing nursing care.

The nurse is performing stroke risk screenings at a hospital open house. The nurse has identified four clients who might be at risk for a stroke. Which client is likely at the highest risk for a hemorrhagic stroke? White female, age 60, with history of excessive alcohol intake White male, age 60, with history of uncontrolled hypertension Black male, age 60, with history of diabetes Black male, age 50, with history of smoking

White male, age 60, with history of uncontrolled hypertension Explanation: Uncontrolled hypertension is the primary cause of a hemorrhagic stroke. Control of hypertension, especially in individuals over 55 years of age, clearly reduces the risk for hemorrhagic stroke. Additional risk factors are increased age, male gender, and excessive alcohol intake. Another high-risk group includes blacks, where the incidence of first stroke is almost twice that as in Caucasians.

A nurse is performing a neurologic assessment on a client. The nurse observes the client's tongue for symmetry, tremors, and strength, and assesses the client's speech. Which cranial nerve is the nurse assessing? IV IX VI XII

XII Explanation: Cranial nerve XII, the hypoglossal nerve, controls tongue movements involved in swallowing and speech. The tongue should be midline, symmetrical, and free from tremors and fasciculations. The nurse tests tongue strength by asking the client to push his tongue against his cheek as the nurse applies resistance. To test the client's speech, the nurse may ask him to repeat the sentence, "Round the rugged rock that ragged rascal ran." The trochlear nerve (IV) is responsible for extraocular movement (inferior medial). The glossopharyngeal nerve (IX) is responsible for swallowing movements and throat sensations. It's also responsible for taste in the posterior third of the tongue. The abducent nerves (VI) are responsible for lateral extraocular movements.

Which are possible long-term complications of spinal cord injury? Select all that apply. respiratory arrest areflexia autonomic dysreflexia respiratory infection

autonomic dysreflexia respiratory infection Explanation: Autonomic dysreflexia and respiratory infection are long-term complications of spinal cord injury. Respiratory arrest and spinal shock (areflexia) are immediate complications of spinal cord injury.

A client with a spinal cord injury and subsequent urine retention receives intermittent catheterization every 4 hours. The average catheterized urine volume has been 550 ml. The nurse should plan to: increase the frequency of the catheterizations. insert an indwelling urinary catheter. place the client on fluid restrictions. use a condom catheter instead of an invasive one.

increase the frequency of the catheterizations. Explanation: As a rule of practice, if intermittent catheterization for urine retention typically yields 500 ml or more, the frequency of catheterization should be increased. Indwelling catheterization is less preferred because of the risk of urinary tract infection and the loss of bladder tone. Fluid restrictions aren't indicated in this case; the problem isn't overhydration, rather it's urine retention. A condom catheter doesn't help empty the bladder of the client with urine retention.

For a client with suspected increased intracranial pressure (ICP), an appropriate respiratory goal is to: prevent respiratory alkalosis. lower arterial pH. promote carbon dioxide elimination. maintain partial pressure of arterial oxygen (PaO2) above 80 mm Hg.

promote carbon dioxide elimination. Explanation: The goal of treatment for ICP is to prevent acidemia by eliminating carbon dioxide because an acid environment in the brain causes cerebral vessels to dilate and therefore increases ICP. Preventing respiratory alkalosis and lowering arterial pH may bring about acidosis, an undesirable condition in this client. It isn't necessary to maintain a PaO2 as high as 80 mm Hg; 60 mm Hg will adequately oxygenate most clients.

A client recently experienced a stroke with accompanying left-sided paralysis. His family voices concerns about how to best interact with him. They report the client doesn't seem aware of their presence when they approach him on his left side. What advice should the nurse give the family? "The client is feeling an emotional loss. He'll eventually start acknowledging you on his left side." "The client is unaware of his left side. You should approach him on the right side." "The client is unaware of his left side. You need to encourage him to interact from this side." "This condition is temporary."

"The client is unaware of his left side. You should approach him on the right side." Explanation: The client is experiencing unilateral neglect and is unaware of his left side. The nurse should advise the family to approach him on his unaffected (right) side. Approaching the client on the affected side would be counterproductive. It's too premature to make the determination whether this condition will be permanent.

Which interventions are appropriate for a client with increased intracranial pressure (ICP)? Select all that apply. Administering prescribed antipyretics Elevating the head of the bed to 90 degrees Maintaining aseptic technique with an intraventricular catheter Encouraging deep breathing and coughing every 2 hours Frequent oral care

Administering prescribed antipyretics Maintaining aseptic technique with an intraventricular catheter Frequent oral care Explanation: Controlling fever is an important intervention for a client with increased ICP because fevers can cause an increase in cerebral metabolism and can lead to cerebral edema. Antipyretics are appropriate to control a fever. It is imperative that the nurse use aseptic technique when caring for the intraventricular catheter because of its risk for infection. Oral care should be provided frequently because the client is likely to be placed on a fluid restriction and will have dry mucous membranes. A nondrying oral rinse may be used. Coughing should be discouraged in a client with increased ICP because it increases intrathoracic pressure, and thus ICP. Unless contraindicated, the head of the bed should be elevated to 30 to 45 degrees and in a neutral position to allow for venous drainage.

A client who recently experienced a stroke tells the nurse that he has double vision. Which nursing intervention is most appropriate? Encourage the client to close his eyes. Alternatively patch one eye every 2 hours. Turn out the lights in the room. Instill artificial tears.

Alternatively patch one eye every 2 hours. Explanation: Patching one eye at a time relieves diplopia (double vision). Closing the eyes and making the room dark aren't the most appropriate options because they deprive the client of sensory input. Artificial tears relieve eye dryness but don't treat diplopia.

A client is diagnosed with a brain tumor. As the nurse assists the client from the bed to a chair, the client begins having a generalized seizure. Which action should the nurse take first? Initiate the code team response. Put a padded tongue blade into the client's mouth and restrain his extremities. Record the type of seizure and the time that it occurred. Assist the client to the floor, in a side-lying position, and protect him with linens.

Assist the client to the floor, in a side-lying position, and protect him with linens. Explanation: The nurse should protect the client from injury by assisting him to the floor, in a side-lying position, and protect him from harm by padding the floor with bed linens. Initiating a response from the code team isn't necessary because seizures are self-limiting. As long as the client's airway is protected, his cardiopulmonary status isn't affected. The nurse shouldn't force anything into the client's mouth during a seizure; doing so may cause injury. Documenting seizure activity is important, but it doesn't take priority over client safety.

A client is scheduled for an EEG. The client asks about any diet-related prerequisites before the EEG. Which diet-related advice should the nurse provide to the client? Avoid eating food at least 8 hours before the test. Include an increased amount of minerals in the diet. Decrease the amount of minerals in the diet. Avoid taking sedative drugs or drinks that contain caffeine for at least 8 hours before the test.

Avoid taking sedative drugs or drinks that contain caffeine for at least 8 hours before the test. The client is advised to refrain from taking sedative drugs or consuming drinks that contain caffeine at least 8 hours before the test because these may interfere with the EEG results. The client is not advised to increase or decrease the intake of minerals in the diet or to avoid eating food 8 hours before the test.

A 76-year-old male client is brought to the clinic by his daughter. The daughter states that her father has had two transient ischemic attacks (TIAs) in the past week. The physician orders carotid angiography, and the report reveals that the carotid artery has been narrowed by atherosclerotic plaques. What treatment option does the nurse expect the physician to offer this client to increase blood flow to the brain? Select all that apply. Balloon angioplasty of the carotid artery followed by stent placement Removal of the carotid artery Percutaneous transluminal coronary artery angioplasty Carotid endarterectomy Administration of tissue plasminogen activator

Balloon angioplasty of the carotid artery followed by stent placement Carotid endarterectomy Explanation: If narrowing of the carotid artery by atherosclerotic plaques is the cause of the TIAs, a carotid endarterectomy (surgical removal of atherosclerotic plaque) could be performed. A balloon angioplasty of the carotid artery, a procedure similar to a percutaneous transluminal coronary artery angioplasty, may be performed alternatively to dilate the carotid artery and increase blood flow to the brain, followed by stent placement. The other options are not options to increase blood flow through the carotid artery to the brain.

A 33-year-old client presents at the clinic with reports of weakness, incoordination, dizziness, and loss of balance. The client is hospitalized and diagnosed with MS. What sign or symptom, revealed during the initial assessment, is typical of MS? Diplopia, history of increased fatigue, and decreased or absent deep tendon reflexes Flexor spasm, clonus, and negative Babinski reflex Blurred vision, intention tremor, and urinary hesitancy Hyperactive abdominal reflexes and history of unsteady gait and episodic paresthesia in both legs

Blurred vision, intention tremor, and urinary hesitancy Explanation: Optic neuritis, leading to blurred vision, is a common early sign of MS, as is intention tremor (tremor when performing an activity). Nerve damage can cause urinary hesitancy. In MS, deep tendon reflexes are increased or hyperactive. A positive Babinski reflex is found in MS. Abdominal reflexes are absent with MS.

Which of the following is the earliest sign of increasing intracranial pressure (ICP)? Change in level of consciousness (LOC) Seizures Restlessness Pupil changes

Change in level of consciousness (LOC) Explanation: The earliest sign of increasing ICP is a change in LOC. Any changes in LOC should be reported immediately. Seizures, restlessness, and pupil changes may occur, but these are not the earliest signs.

Which is a correct rationale for encouraging a client with otitis externa to eat soft foods? Chewy foods, such as red meat, may react with prescribed analgesics and antibiotics. Chewing may cause discomfort. Chewing may lead to further complications, such as otitis media. Chewing may cause excessive drainage.

Chewing may cause discomfort. Explanation: The nurse encourages a client with otitis externa to eat soft foods or consume nourishing liquids because chewing may cause discomfort. Chewing will not react with the prescribed medications or cause complications such as otitis media and excessive drainage.

Which assessment finding is most important in determining nursing care for a client with bacterial meningitis? Select all that apply. Cloudy cerebral spinal fluid Pain and stiffness of the extremities Purpura of hands and feet Low white blood cell (WBC) count Low red blood cell (RBC) count Low antidiuretic hormone (ADH) levels

Cloudy cerebral spinal fluid Purpura of hands and feet Explanation: The cerebral spinal fluid (CSF) will be cloudy if bacterial meningitis is the causative agent. Purpura indicates a serious complication of bacterial meningitis (disseminated intravascular coagulation) is occurring and may place the client at risk for amputation of those parts. Pain and stiffness of the extremities is not indicative of meningitis. A rise in RBCs, WBCs, and ADH would be expected.

Which insult or abnormality can cause an ischemic stroke? Cocaine use Arteriovenous malformation Trauma Intracerebral aneurysm rupture

Cocaine use Explanation: Cocaine is a potent vasoconstrictor and may result in a life-threatening reaction, even with the individual's first use of the drug. Arteriovenous malformations, trauma, and intracerebral aneurysm rupture are associated with hemorrhagic stroke.

The nurse is offering suggestions regarding reproductive options to a husband and paraplegic wife. Which option is most helpful? Adoption is an option to complete your family but not put your life in jeopardy. Conception is not impaired; the birth process is determined with the physician. Birth via surrogate is best because your baby can be implanted in another woman. Sterilization is best; it would be difficult to care for a baby in your condition.

Conception is not impaired; the birth process is determined with the physician. Explanation: The nurse's role is to provide facts without inserting personal opinions. The fact is that the woman can conceive and bear children. Suggesting adoption, a surrogate, and sterilization is not appropriate. Providing information on that suggestion is appropriate.

Which type of brain injury is characterized by a loss of consciousness associated with stupor and confusion? Concussion Contusion Diffuse axonal injury Intracranial hemorrhage

Contusion Explanation: Other characteristics can include tissue alteration and neurologic deficit without hematoma formation, alteration in consciousness without localizing signs, and hemorrhage into the tissue that varies in size and is surrounded by edema. The effects of injury (hemorrhage and edema) peak after about 18 to 36 hours. A concussion is a temporary loss of neurologic function with no apparent structural damage. A diffuse axonal injury involves widespread damage to the axons in the cerebral hemispheres, corpus callosum, and brainstem. An intracranial hemorrhage is a collection of blood that develops within the cranial vault.

A 58-year-old construction worker fell from a 25-foot scaffolding and incurred a closed head injury as a result. As his intracranial pressure continues to increase, the potential of herniation also increases. If the brain herniates, which of the following are potential consequences? Choose all correct options. Death Permanent neurologic dysfunction Impaired cellular activity Insomnia Seizures

Death Permanent neurologic dysfunction Impaired cellular activity Explanation: As increased ICP progresses, the consequences include impaired cellular activity, temporary or permanent neurologic dysfunction, or death.

The nurse is discharging home a client who had a stroke. The client has a flaccid right arm and leg and is experiencing urinary incontinence. The nurse makes a referral to a home health nurse because of an awareness of what common client response to a change in body image? Confusion Uncertainty Depression Disassociation

Depression Explanation: Depression is a common and serious problem in the client who has had a stroke. It can result from a profound disruption in his or her life and changes in total function, leaving the client with a loss of independence. The nurse needs to encourage the client to verbalize feelings to assess the effect of the stroke on self-esteem. Confusion, uncertainty, and disassociation are not the most common client response to a change in body image, although each can occur in some clients.

A client with a suspected brain tumor is scheduled for a computed tomography (CT) scan. What should the nurse do when preparing the client for this test? Immobilize the neck before the client is moved onto a stretcher. Determine whether the client is allergic to iodine, contrast dyes, or shellfish. Place a cap over the client's head. Administer a sedative as ordered.

Determine whether the client is allergic to iodine, contrast dyes, or shellfish. Explanation: Because CT commonly involves use of a contrast agent, the nurse should determine whether the client is allergic to iodine, contrast dyes, or shellfish. Neck immobilization is necessary only if the client has a suspected spinal cord injury. Placing a cap over the client's head may lead to misinterpretation of test results; instead, the hair should be combed smoothly. The physician orders a sedative only if the client can't be expected to remain still during the CT scan.

The nurse is caring for a client hospitalized after a motor vehicle accident. The client has a comorbidity of Parkinson's disease. Why should the nurse closely monitor the condition and the drug regimen of a client with Parkinson's disease? Drugs administered may not cause the requisite therapeutic effect. Clients take an assortment of different drugs. Clients generally do not adhere to the drug regimen. Drugs administered may cause a wide variety of adverse effects.

Drugs administered may cause a wide variety of adverse effects. Explanation: Drugs administered for Parkinsonism may cause a wide variety of adverse effects, which requires careful observation of the client. Over time, clients may respond less and less to their standard drug therapy and have more frequent "off episodes" of hypomobility. As a result, the nurse should administer the drugs closely to the schedule. Generally, a single drug called levodopa is administered to clients with Parkinson's disease. It is also not true that drugs may not cause the requisite therapeutic effect or such clients do not adhere to the drug regimen.

The nurse is evaluating the transmission of a report from a paramedic unit to the emergency room. The medic reports that a client is unconscious with edema of the head and face and Battle's sign. What clinical picture would the nurse anticipate? Edema to the head and a blackened eye Edema to the head with a large scalp laceration Edema to the head with fixed pupils Edema to the head with bruising of the mastoid process

Edema to the head with bruising of the mastoid process Explanation: Battle's sign is the presence of bruising of the mastoid process behind the ear. It is not related to periorbital bleeding, lacerations, or fixed pupils.

A client arrives at the emergency department complaining of extreme muscle weakness after minimal effort. The physician suspects myasthenia gravis. Which drug will be used to test for this disease? Ambenonium (Mytelase) Pyridostigmine (Mestinon) Edrophonium (Tensilon) Carbachol (Carboptic)

Edrophonium (Tensilon) Explanation: Edrophonium temporarily blocks the breakdown of acetylcholine, thus increasing acetylcholine level in the blood, and relieves weakness. Because of its short duration of action, edrophonium is the drug of choice for diagnosing myasthenia gravis. It's also used to differentiate myasthenia gravis from cholinergic toxicity. Ambenonium is used as an antimyasthenic. Pyridostigmine serves primarily as an adjunct in treating severe anticholinergic toxicity; it's also an antiglaucoma agent and a miotic. Carbachol reduces intraocular pressure during ophthalmologic procedures; topical carbachol is used to treat open-angle and closed-angle glaucoma.

A client is undergoing testing to confirm a diagnosis of myasthenia gravis. The nurse explains that a diagnosis is made if muscle function improves after the client receives an IV injection of a medication. What is the medication the nurse tells the client he'll receive during this test? Cyclosporine (Sandimmune) Edrophonium (Tensilon) Immunoglobulin G (Iveegam EN) Azathioprine (Imuran)

Edrophonium (Tensilon) Explanation: The most useful and reliable diagnostic test for myasthenia gravis is the edrophonium (Tensilon) test. Within 30 to 60 seconds after injection of edrophonium, most clients with myasthenia gravis will demonstrate a marked improvement in muscle tone that lasts about 4 to 5 minutes. Cyclosporine, an immunosuppressant, is used to treat myasthenia gravis, not to diagnose it. Immunoglobulin G is used during acute relapses of the disorder. Azathioprine is an immunosuppressant that's sometimes used to control myasthenia gravis symptoms.

A client is hospitalized with Guillain-Barré syndrome. Which nursing assessment finding is most significant? Warm, dry skin Urine output of 40 ml/hour Soft, nondistended abdomen Even, unlabored respirations

Even, unlabored respirations Explanation: A characteristic feature of Guillain-Barré syndrome is ascending weakness, which usually begins in the legs and progresses upward to the trunk, arms, and face. Respiratory muscle weakness, evidenced by even, unlabored respirations, is a particularly dangerous effect of this disease progression because it may lead to respiratory failure and death. Therefore, although warm, dry skin; urine output of 40 ml/hour; and a soft, nondistended abdomen are pertinent assessment data, those related to respiratory function and status are most significant.

A nurse is communicating with a client who has aphasia after having a stroke. Which action should the nurse take? Use one long sentence to say everything that needs to be said. Keep the television on while she speaks. Talk in a louder than normal voice. Face the client and establish eye contact.

Face the client and establish eye contact. Explanation: When speaking with a client who has aphasia, the nurse should face the client and establish eye contact. The nurse should use short phrases, not one long sentence, and give the client time between phrases to understand what is being said. Keeping extraneous and background noise such as the television to a minimum helps the client concentrate on what is being said. It isn't necessary to speak in a louder or softer voice than normal.

A patient has a severe neurologic impairment from a head trauma. What does the nurse recognize is the type of posturing that occurs with the most severe neurologic impairment? Decerebrate Decorticate Flaccid Rigid

Flaccid Explanation: LOC, a sensitive indicator of neurologic function, is assessed based on the criteria in the Glasgow Coma Scale: eye opening, verbal response, and motor response (Barlow, 2012). The patient's responses are rated on a scale from 3 to 15. A score of 3 indicates severe impairment of neurologic function, brain death, or pharmacologic inhibition of the neurologic response. A score of 15 indicates that the patient is fully responsive (see Chapter 68). An inappropriate or nonpurposeful response is random and aimless. Posturing may be decorticate or decerebrate (Fig. 66-1; see also Chapter 65). The most severe neurologic impairment results in flaccidity. The motor response cannot be elicited or assessed when the patient has been administered pharmacologic paralyzing agents (i.e., neuromuscular blocking agents).

In a spinal cord injury, neurogenic shock develops due to loss of the autonomic nervous system functioning below the level of the lesion. Which of the following indicators of neurogenic shock would the nurse expect to find? Select all that apply. (6 options) Hypotension Tachycardia Venous pooling Diaphoresis Tachypnea Hypothermia

Hypotension Venous pooling Tachypnea Hypothermia Explanation: The vital organs are affected in a spinal cord injury, causing the blood pressure and heart rate to decrease. This loss of sympathetic innervation causes a variety of other clinical manifestations, including a decrease in cardiac output, venous pooling in the extremities, and peripheral vasodilation resulting in mild hypotension, bradycardia, and warm skin. In addition, the patient does not perspire on the paralyzed portions of the body because sympathetic activity is blocked; therefore, close observation is required for early detection of an abrupt onset of fever.

A client diagnosed with migraine headaches asks the nurse what he can do to help control the headaches and minimize the number of attacks he is having. What instructions should the nurse give this client? Identify and avoid factors that precipitate or intensify an attack. Keep a record of activities following an attack. When an attack occurs stay in a brightly lit area. Write down any adverse drug effects.

Identify and avoid factors that precipitate or intensify an attack. Explanation: The nurse instructs the client to follow the indications and dosage regimen for medication, notify the physician of any adverse drug effects, and identify and avoid factors that precipitate or intensify an attack. Keeping a food diary may help identify foods that trigger attacks. The client can keep a record of the attacks, including activities before the attack, and environmental or emotional circumstances that appear to bring on the attack. The client should lie down in a darkened room, and avoid noise and movement when an attack occurs, if that is possible.

A client arrives at the ED via ambulance following a motorcycle accident. The paramedics state the client was found unconscious at the scene but briefly regained consciousness during transport to the hospital. Upon initial assessment, the client's GCS score is 7. The nurse anticipates which action? Immediate craniotomy An order for a head computed tomography scan Intubation and mechanical ventilation IV administration of propofol

Immediate craniotomy Explanation: The client is experiencing an epidural hematoma. An epidural hematoma is considered an extreme emergency; marked neurologic deficit or even respiratory arrest can occur within minutes. Treatment consists of making openings through the skull (burr holes) to decrease intracranial pressure (ICP) emergently, remove the clot, and control the bleeding. A craniotomy may be required to remove the clot and control the bleeding. Epidural hematomas are often characterized by a brief loss of consciousness followed by a lucid interval in which the client is awake and conversant. During this lucid interval, the expanding hematoma is compensated for by rapid absorption of cerebrospinal fluid and decreased intravascular volume, both of which help to maintain the ICP within normal limits. When these mechanisms can no longer compensate, even a small increase in the volume of the blood clot produces a marked elevation in ICP. The client then becomes increasingly restless, agitated, and confused as the condition progresses to coma.

A patient has expressive speaking aphasia after having a stroke. Which portion of the brain does the nurse know has been affected? Temporal lobe Inferior posterior frontal areas Posterior frontal area Parietal-occipital area

Inferior posterior frontal areas Explanation: A deficiency in language function is called aphasia. Expressive speaking aphasia is associated with injury to the inferior posterior frontal areas, auditory receptive aphasia with the temporal lobe, expressive writing aphasia with the posterior frontal area, and visual receptive aphasia with the parietal and occipital areas.

A nurse is caring for an older client who has had a hemorrhagic stroke. The client has exhibited impulsive behavior and, despite reminders from the nurse, doesn't recognize his limitations. Which priority measure should the nurse implement to prevent injury? Encourage the client to do as much as possible without assistance, and to use the call light only in emergencies. Install a bed alarm to remind the client to ask for assistance and to alert staff that the client is getting out of bed. Encourage the family to reprimand the client if he doesn't ask for help with transfers and mobility. Ask a physician to order a vest and wrist restraints.

Install a bed alarm to remind the client to ask for assistance and to alert staff that the client is getting out of bed. Explanation: The bed alarm will alert staff that the client is attempting to transfer, so they can come to assist. The nurse shouldn't encourage the family to reprimand the client. Instead, the nurse should ask the family to encourage the client to request assistance. The nurse should encourage the client to use the call light in all situations, not just emergencies. A vest and wrist restraints aren't appropriate unless less-restrictive measures have failed and the client is a danger to himself or others.

A nurse is assisting during a lumbar puncture. How should the nurse position the client for this procedure? Prone, with the head turned to the right Supine, with the knees raised toward the chest Lateral recumbent, with chin resting on flexed knees Lateral, with right leg flexed

Lateral recumbent, with chin resting on flexed knees Explanation: To maximize the space between the vertebrae, the client is placed in a lateral recumbent position with knees flexed toward the chin. The needle is inserted between L4 and L5. The other positions wouldn't allow as much space between L4 and L5.

An 83-year-old woman suffers a stroke at home and is hospitalized for treatment and management. Which of the following diagnostic procedures would be best to visualize the extent of damage? Magnetic resonance imaging (MRI) Diffusion-weighted imaging (DWI) Magnetic resonance angiography (MRA) Computed tomography (CT)

Magnetic resonance angiography (MRA) An MRA allows separate visualization of the cerebral vasculature and can be used in place of an MRI.

A client has been admitted to the ICU after being recently diagnosed with an aneurysm and the client's admission orders include specific aneurysm precautions. What nursing action will the nurse incorporate into the client's plan of care? Elevate the head of the bed to 75 degrees. Maintain the client on complete bed rest. Administer enemas when the client is constipated. Leg exercises to prevent deep vein thrombosis

Maintain the client on complete bed rest. Explanation: Cerebral aneurysm precautions are implemented for the client with a diagnosis of aneurysm to provide a nonstimulating environment, prevent increases in ICP, and prevent further bleeding. The client is placed on immediate and absolute bed rest in a quiet, nonstressful environment because activity (such as leg exercises), pain, and anxiety elevate BP, which increases the risk for bleeding. Visitors, except for family, are restricted. The head of the bed is elevated 30 degrees to promote venous drainage and decrease ICP. Some neurologists, however, prefer that the client remains flat to increase cerebral perfusion. No enemas are permitted, but stool softeners and mild laxatives are prescribed.

There is a high risk for ineffective coping in a client with a recent spinal cord injury. Which nursing interventions will assist the client with this process? Select all that apply. Assist the client in accepting the severity of deficits. Offer encouragement as the client makes progress. Involve the client actively in self care. Reassure the client by stating, "Everything is going to be all right."

Offer encouragement as the client makes progress. Involve the client actively in self care. Explanation: Encouragement can contribute to the client's resolve to put forth continued effort and involving the client in care will assist with effective coping. Until the initial trauma and swelling have resolved, assumptions about outcome are premature. False reassurance is a common barrier to therapeutic communication.

The nursing instructor is teaching the senior nursing class about neuromuscular disorders. When talking about Multiple Sclerosis (MS) what diagnostic finding would the instructor list as being confirmatory of a diagnosis of MS? An elevated acetylcholine receptor antibody titer Episodes of muscle fasciculations IV administration of edrophonium Oligoclonal bands

Oligoclonal bands Explanation: Electrophoresis of the CSF, a technique for electrically separating and identifying proteins, demonstrates abnormal immunoglobulin G bands, described as oligoclonal bands. An elevated acetylcholine receptor antibody titer and IV administration of edrophonium are diagnostic of Mysthenia Gravis. Episodes of muscle fasciculations are characteristic of ALS.

The nurse is caring for a client with a traumatic brain injury who has developed increased intracranial pressure resulting in syndrome of inappropriate antidiuretic hormone (SIADH). While assessing this client, the nurse expects which of the following findings? Excessive urine output and decreased urine osmolality Oliguria and serum hyponatremia Oliguria and serum hyperosmolarity Excessive urine output and serum hyponatremia

Oliguria and serum hyponatremia Explanation: SIADH is the result of increased secretion of antidiuretic hormone (ADH). The client becomes volume overloaded, urine output diminishes, and serum sodium concentration becomes dilute.

The client has been diagnosed with aphasia after suffering a stroke. What can the nurse do to best make the client's atmosphere more conducive to communication? Provide a board of commonly used needs and phrases. Have the client speak to loved ones on the phone daily. Help the client complete his or her sentences as needed. Speak in a loud and deliberate voice to the client.

Provide a board of commonly used needs and phrases. Explanation: The inability to talk on the telephone or answer a question or exclusion from conversation causes anger, frustration, fear of the future, and hopelessness. A common pitfall is for the nurse or other health care team member to complete the thoughts or sentences of the client. This should be avoided because it may cause the client to feel more frustrated at not being allowed to speak and may deter efforts to practice putting thoughts together and completing a sentence. The client may also benefit from a communication board, which has pictures of commonly requested needs and phrases. The board may be translated into several languages.

A nurse is planning care for a client who experienced a stroke in the right hemisphere of his brain. What should the nurse do? Anticipate the client will exhibit some degree of expressive or receptive aphasia. Place the wheelchair on the client's left side when transferring him into a wheelchair. Provide close supervision because of the client's impulsiveness and poor judgment. Support the right arm with a sling or pillow to prevent subluxation.

Provide close supervision because of the client's impulsiveness and poor judgment. Explanation: The primary symptoms of a client who experiences a right-sided stroke are left-sided weakness, impulsiveness, and poor judgment. Aphasia is more commonly present when the dominant or left hemisphere is damaged. When a client has one-sided weakness, the nurse should place the wheelchair on the client's unaffected side. Because a right-sided stroke causes left-sided paralysis, the right side of the body should remain unaffected.

A client has sustained a traumatic brain injury with involvement of the hypothalamus. The health care team is concerned about the complication of diabetes insipidus. Which of the following would be an appropriate nursing intervention to monitor for early signs of diabetes insipidus? Assess frequent vital signs. Reposition frequently. Assess for pupillary response frequently. Record intake and output.

Record intake and output. Explanation: A record of intake and output is maintained for the client with a traumatic brain injury, especially if the client has hypothalamic involvement and is at risk for the development of diabetes insipidus. Excessive output will alert the nurse to possible fluid imbalance early in the process.

Which nursing interventions would be useful for a client with ICP related to bleeding in the brain? Select all that apply. Report changes in neurologic status as soon as a worsening trend is identified. Use the Glasgow Coma Scale to assess neurologic status at least every 2 hours. Follow the physician's orders to increase fluid volume. Avoid any activities that cause a Valsalva maneuver.

Report changes in neurologic status as soon as a worsening trend is identified. Avoid any activities that cause a Valsalva maneuver. Explanation: Effector T cells are killer (cytotoxic) cells.

Which of the following are the immediate complications of spinal cord injury? Respiratory arrest Tetraplegia Spinal shock Paraplegia Autonomic dysreflexia

Respiratory arrest Spinal shock Explanation: Respiratory arrest and spinal shock are the immediate complications of spinal cord injury. Tetraplegia is paralysis of all extremities when there is a high cervical spine injury. Paraplegia occurs with injuries at the thoracic level. Autonomic dysreflexia is a long-term complication of spinal cord injury.

A nurse is providing care to a client with a brain tumor. The client has experienced seizures as a result of the tumor. Which area would be a priority for this client? Safety Self-care Skin care Activity

Safety Explanation: Clients who have seizures are carefully monitored and protected from injury. Therefore, safety is the priority.

A nurse is assessing a client with Parkinson's disease. Which of the following would the nurse expect to find? Gait with the body leaning backward Continuous tremors Muscle flaccidity Slowing of activity

Slowing of activity Explanation: Clients with Parkinson's disease typically manifest bradykinesia (slowing of all active movement), a propulsive, forward leaning gait, tremors that disappear with active movement, and muscle rigidity.

A client with respiratory complications of multiple sclerosis (MS) is admitted to the medical-surgical unit. Which equipment is most important for the nurse to keep at the client's bedside? Sphygmomanometer Padded tongue blade Nasal cannula and oxygen Suction machine with catheters

Suction machine with catheters Explanation: MS weakens the respiratory muscles and impairs swallowing, putting the client at risk for aspiration. To ensure a patent oral airway, the nurse should keep a suction machine and suction catheters at the bedside. A sphygmomanometer is no more important for this client than for any other. A padded tongue blade is an appropriate seizure precaution but shouldn't be used in this client because its large size could cause oral airway obstruction. A nasal cannula and oxygen would be ineffective to ensure adequate oxygen delivery; this client requires a mechanical ventilator.

A client who suffered an ischemic stroke now has disturbed sensory perception. What principle should guide the nurse's care of this client? The client should be approached on the side where visual perception is intact. Attention to the affected side should be minimized in order to decrease anxiety. The client should avoid turning in the direction of the defective visual field to minimize shoulder subluxation. The client should be approached on the opposite side of where the visual perception is intact to promote recovery.

The client should be approached on the side where visual perception is intact. Explanation: Clients with decreased field of vision should first be approached on the side where visual perception is intact. All visual stimuli should be placed on this side. The client can and should be taught to turn the head in the direction of the defective visual field to compensate for this loss. The nurse should constantly remind the client of the other side of the body and should later stand at a position that encourages the client to move or turn to visualize who and what is in the room.

A client who's paralyzed on the left side has been receiving physical therapy and attending teaching sessions about safety. Which behavior indicates that the client accurately understands safety measures related to paralysis? The client leaves the side rails down. The client uses a mirror to inspect the skin. The client repositions only after being reminded to do so. The client hangs the left arm over the side of the wheelchair.

The client uses a mirror to inspect the skin. Explanation: The client demonstrates understanding of safety measures related to paralysis when he uses a mirror to inspect his skin. The mirror enables the client to inspect all areas of the skin for signs of breakdown without the help of staff or family members. The client should keep the side rails up to help with repositioning and to prevent falls. The paralyzed client should take responsibility for repositioning or for reminding the staff to assist with it, if needed. A client with left-side paralysis may not realize that the left arm is hanging over the side of the wheelchair. However, the nurse should call this position to the client's attention because the arm can get caught in the wheel spokes or develop impaired circulation from being in a dependent position for too long.

The nurse is completing the physical assessment of a patient suspected of a neurologic disorder. The patient reports to the nurse that he has recently suffered a head trauma. In such a case, which of the following precautions should the nurse take for the patient? Select all that apply. The nurse should make the patient sit in a chair and then assess his or her head for bleeding or swelling. The nurse should only move the patient's head with the help of an assistant. The nurse should explain the procedure of head assessment to the patient before doing the assessment. The nurse should not move or manipulate the patient's head while assessing for bleeding or swelling.

The nurse should not move or manipulate the patient's head while assessing for bleeding or swelling. The nurse evaluates the patient's body posture and any abnormal position of the head, neck, trunk, or extremities. The nurse carefully examines the head for bleeding, swelling, or wounds. The nurse does not move or manipulate the patient's head during physical assessment, especially if there is a recent history of trauma. The nurse should not make the patient sit on a chair or seek the help of an assistant while doing the head assessment. The nurse need not explain in detail about the procedure of head assessment to the patient.

The nurse is preparing the client for a diagnostic test to evaluate blood flow within intracranial blood vessels. For which test is the nurse preparing the client? Computed tomography Magnetic resonance imaging Transcranial Doppler Cerebral angiography

Transcranial Doppler Transcranial Doppler flow studies are used to study a tumor's blood flow within intracranial blood vessels. Cerebral angiography may be used to study a tumor's blood supply or obtain information about vascular lesions. Magnetic resonance imaging provides information similar to that provided by computed tomography, but with improved tissue contrast, resolution, and anatomic definition, and it examines the lesion in multiple planes.

A client with meningitis has a history of seizures. Which action by the nurse is appropriate while the client is actively seizing? Place a cooling blanket on the client Administer mannitol Turn the client to the side Insert oral airway

Turn the client to the side Explanation: When a client is seizing, the nurse should turn the client to the side and not restrain his or her movements. This helps reduce the potential for aspiration of saliva or stomach contents. An oral airway should not be inserted while the client is actively seizing. An oral airway may be inserted during the aura phase. Anticonvulsants may be administered, but mannitol is an osmotic diuretic, not an anticonvulsant. Applying a cooling blanket while the client is actively seizing could cause harm to the client and is not indicated for seizure activity.


Ensembles d'études connexes

Common Idiom Sentence Fill in the Blank

View Set

ANT 2410 Ch. 6: Race and Ethnicity

View Set

RICCI Peds Review Exam 1 - Chapter 48

View Set

Pharm CH 27- Antipsychotics & Anxiolytics

View Set

Great Gatsby worksheet questions

View Set

Chapter 6: Cultural Transmission( PrincipalsofABehavior ed#3) )

View Set